which is larger??

Поделиться
HTML-код
  • Опубликовано: 2 окт 2024
  • We look at a nice inequality.
    Suggest a problem: forms.gle/ea7P...
    Please Subscribe: www.youtube.co...
    Merch: teespring.com/...
    Personal Website: www.michael-pen...
    Randolph College Math: www.randolphcol...
    Randolph College Math and Science on Facebook: / randolph.science
    Research Gate profile: www.researchga...
    Google Scholar profile: scholar.google...
    If you are going to use an ad-blocker, considering using brave and tipping me BAT!
    brave.com/sdp793
    Buy textbooks here and help me out: amzn.to/31Bj9ye
    Buy an amazon gift card and help me out: amzn.to/2PComAf
    Books I like:
    Sacred Mathematics: Japanese Temple Geometry: amzn.to/2ZIadH9
    Electricity and Magnetism for Mathematicians: amzn.to/2H8ePzL
    Abstract Algebra:
    Judson(online): abstract.ups.edu/
    Judson(print): amzn.to/2Xg92wD
    Dummit and Foote: amzn.to/2zYOrok
    Gallian: amzn.to/2zg4YEo
    Artin: amzn.to/2LQ8l7C
    Differential Forms:
    Bachman: amzn.to/2z9wljH
    Number Theory:
    Crisman(online): math.gordon.edu...
    Strayer: amzn.to/3bXwLah
    Andrews: amzn.to/2zWlOZ0
    Analysis:
    Abbot: amzn.to/3cwYtuF
    How to think about Analysis: amzn.to/2AIhwVm
    Calculus:
    OpenStax(online): openstax.org/s...
    OpenStax Vol 1: amzn.to/2zlreN8
    OpenStax Vol 2: amzn.to/2TtwoxH
    OpenStax Vol 3: amzn.to/3bPJ3Bn
    My Filming Equipment:
    Camera: amzn.to/3kx2JzE
    Lense: amzn.to/2PFxPXA
    Audio Recorder: amzn.to/2XLzkaZ
    Microphones: amzn.to/3fJED0T
    Lights: amzn.to/2XHxRT0
    White Chalk: amzn.to/3ipu3Oh
    Color Chalk: amzn.to/2XL6eIJ

Комментарии • 172

  • @Carhill
    @Carhill 3 года назад +118

    9:50 - that minus sign added in post scared me then... thought a mass array of pixels had died on my monitor.

  • @graysonking16
    @graysonking16 2 года назад +17

    Pre-watch guess: a^a + b^b is larger because either a or b is larger, and exponentials grow very quickly, so being able to raise the larger number to itself probably makes up for the fact that you would get to raise the smaller number to the larger number in the opposing case.

  • @kajamix
    @kajamix 3 года назад +97

    Much simpler proof: We want to prove a^b + b^a

  • @mathunt1130
    @mathunt1130 3 года назад +4

    As he said, wlog b>a, then fix a. Examine the function f(b)=a^a+b^b-a^b-b^a, and compute f'(b) and examine the sign. It's late at night and this was the first thing I thought of.

  • @ShinySwalot
    @ShinySwalot 3 года назад +46

    It's very intuitive too, either you're doing the highest value to the highest power or the highest value to the lowest power.

    • @vinc17fr
      @vinc17fr 3 года назад +1

      I was wondering something like that and thinking about a generalization: Let 0 < a ≤ b and 0 < u ≤ v. Which is larger a^u + b^v or a^v + b^u? I suppose that it is a^u + b^v, but I haven't tried to prove it.

    • @ShinySwalot
      @ShinySwalot 3 года назад

      @@vinc17fr Hmmm, that's actually a really interesting idea, I wonder if that's enough restriction in order to generalize it though.

    • @vinc17fr
      @vinc17fr 3 года назад +1

      @@ShinySwalot Actually, that's not true, at least for b < 1. Here's a counter-example: (1/3)^2 + (1/2)^1000 < (1/3)^1000 + (1/2)^2.

    • @vinc17fr
      @vinc17fr 3 года назад +1

      @@ShinySwalot I think that one needs to consider the different cases, something like whether the values are less or greater than 1. Using u = a and v = b like in the original problem makes these conditions more restrictive.

    • @ShinySwalot
      @ShinySwalot 3 года назад

      @@vinc17fr Or just make it over the natural numbers instead? Just proposing something lol

  • @timurpryadilin8830
    @timurpryadilin8830 3 года назад +34

    Homework: let f(t) = (ln t- ln a) / (t-a). Then f'(t) = ((t-a)/t-lnt+lna) / (t-a)^2 has the same sign as 1-a/t + ln(a/t). Now let g(y) =1-y+lny, so that f'(t) = g(a/t) . g'(y) = 1/y-1 > 0 when ya. g(1)=0, so g(y) < 0 for y

    • @karangupta1825
      @karangupta1825 3 года назад +3

      Try this :
      ((a(subi))^2)) + ((a(subi))+1)^2 = ((b(subi))^2)
      &
      ((x(subi))^2) -1 = 2((y(subi))^2)
      Edit:
      Then prove:
      (x(subi))+(y(subi)) = (b(sub(i+1)))
      (x(subi))-(y(subi)) = (b(sub(i)))

    • @kongkong5
      @kongkong5 3 года назад +1

      If you let f(t) = ln t, much easier

    • @johnnath4137
      @johnnath4137 3 года назад

      (ln1 - lna)/(1 - a) ≤ (lnb - lna)/(b - a) ↔ -blna + alna ≤ lnb - lna - alnb + alna ↔ -blna ≤ lnb - lna - alnb ↔ ln(ba^b/ab^a) ≥ 0. But (ba^b/ab^a) ≥ (ba^a/ab^a) = (b/a)^(1 - a) ≥ 1 → ln(ba^b/ab^a) ≥ ln(ba^a/ab^a) ≥ 0 → (ln1 - lna)/(1 - a) ≤ (lnb - lna)/(b - a).

  • @TheSmilodon2
    @TheSmilodon2 3 года назад

    i took a^b + b^a < b^b + a^a
    i noted c = b-a (with presumption b > a )
    we have: a^a*a^c + b^a < b^a*b^c + a^a => a^a*(a^c-1) < b^a*(b^c-1) => (a/b)^a < (b^c-1)/(a^c-1)
    since a

  • @goodplacetostop2973
    @goodplacetostop2973 3 года назад +13

    12:13 Homework
    12:37 Good place to stop

  • @MrKinyodude
    @MrKinyodude 3 года назад +1

    Michael, you are right about skin tones ... white balance is quite off, whole scene is reddish. If camera cannot compensate, consider changing light source. Great math as usual!

  • @maxwellsequation4887
    @maxwellsequation4887 3 года назад +23

    How is there comments from 3 weeks ago when the video is just out

    • @fix5072
      @fix5072 3 года назад +4

      channel members have early access to the videos

    • @MichaelPennMath
      @MichaelPennMath  3 года назад +47

      This is not true! I upload ahead of time and leave videos unlisted. I am not sure how people find them...

    • @fix5072
      @fix5072 3 года назад +2

      @@MichaelPennMath Well I'm sorry then. As many RUclipsrs do so I thought it woulb be the same with You since its a nice way to reward channel members.

    • @goodplacetostop2973
      @goodplacetostop2973 3 года назад +21

      @@MichaelPennMath Here's the trick... Go on playlists page, sort them by last video added and there we go. In some playlists, there are some unlisted videos but NOT private which means you can watch them even if they're not officially published.

    • @abhisheknanda85
      @abhisheknanda85 3 года назад +7

      @@MichaelPennMath Adding an unlisted video into a public playlist effectively makes the video public too.

  • @marioytambor
    @marioytambor 3 года назад +3

    On the last part, could we just divide by b-a, since it's positive but bigger than 0? I feel then the homework part is easier ( because adding log(t)/(b-a) and differentiating)

  • @marcrenard515
    @marcrenard515 3 года назад +3

    This two quantities are rate increase of f:x->ln(x), they represant the slope of secant to the curve. Since ln(x) is concave ( f''(x) =-1/x²(ln(x)-ln(a)) /(x-a) is decreasing. We have b

  • @bryguy6651
    @bryguy6651 3 года назад +28

    This is very similar to the Rearrangement Inequality!

    • @mehmeterciyas6844
      @mehmeterciyas6844 3 года назад +6

      Hi from future!

    • @arnabchatterjee4847
      @arnabchatterjee4847 3 года назад +2

      @@mehmeterciyas6844 Hi I am from past

    • @sirlight-ljij
      @sirlight-ljij 3 года назад +1

      Indeed it is. There is a generalisation of this inequality for arbitrary functions -- if we have x1

  • @laurensiusfabianussteven6518
    @laurensiusfabianussteven6518 3 года назад +14

    I thought there will be mean-value theorem...

  • @MrMetrizable
    @MrMetrizable 3 года назад +6

    -ln x is convex

  • @sevdanaamani3266
    @sevdanaamani3266 3 года назад +1

    Convex function and its done.QED

  • @keithmasumoto9698
    @keithmasumoto9698 3 года назад +14

    Proof by contradiction: Say -lna/(1-a) > (lnb-lna)/(b-a). Then doing some cross multiplication, we arrive at lna/(1-a)>lnb/(1-b) which is not true for a

    • @megauser8512
      @megauser8512 3 года назад +1

      Yes, you are correct, since: -lna / (1-a) > (lnb-lna) / (b-a) implies that -lna * (b-a) > (lnb-lna) * (1-a), which leads to -b*lna+a*lna > lnb-lna-a*lnb+a*lna, so -b*lna > lnb-lna-a*lnb, so lna-b*lna > lnb-a*lnb, so lna * (1-b) > lnb * (1-a), so lna/(1-a) > lnb/(1-b),
      but since 0 < a < b < 1, then -1 < -b < -a < 0 and lna < lnb < 0, so 0 < -lnb < -lna, so we also have:
      0 < 1-b < 1-a < 1, so 1 < 1/(1-a) < 1/(1-b), so -1/(1-b) < -1/(1-a) < -1, so -lnb/(1-b) < -lnb/(1-a) < -lna/(1-a),
      so lna/(1-a) < lnb/(1-b), which is a contradiction with the above.

  • @manjunathbhat6934
    @manjunathbhat6934 3 года назад +5

    A simpler approach : Let's scale down a and b wrto a. Let a = 1, b /a = t, t is a positive real number. Hence X = a^b + b^a = 1 + t, and Y = a^a + b^b = 1 + t^t. Compare X and Y, implies comparing t and t^t, implies comparing log(t) and t*log(t). t^t > t for all positive real t, as log(t) * (t - 1) > 0 for all positive real t. Hence Y > X.

    • @jebooiii6462
      @jebooiii6462 3 года назад

      That's really nice

    • @vinc17fr
      @vinc17fr 3 года назад +2

      But how do you prove that you can scale down a and b? Since they appear also in exponent, this is not obvious.

    • @Notthatkindofdr
      @Notthatkindofdr 3 года назад +3

      @@vinc17fr Agreed. There is no way to "scale" the problem to look like this. This solution really only considers the case that a=1.

  • @charlottedarroch
    @charlottedarroch 3 года назад +5

    This inequality reminded me of the rearrangement inequality and in fact a generisation of this inequality is very similar to the rearrangment inequality. The inequality in the video can be extended to the result: If x_1 >= x_2 >= ... >= x_n > 0 and y_1 >= y_2 >= ... >= y_n > 0, then x_1^y_1+x_2^y_2+...+x_n^y_n >= x_1^y_p(1)+x_2^y_p(2)+...+x_n^y_p(n) for any permutation p on the set {1,2,...,n}.
    This is proved by induction using exactly the same argument as is used to prove the rearrangement inequality, starting from the base case that if x_1 >= x_2 > 0 and y_1 >= y_2 > 0, then x_1^y_1+x_2^y^2 >= x_1^y_2+x_2^y_1.
    The base case can be proved by first observing the following equivalences:
    x_1^y_1+x_2^y^2 >= x_1^y_2+x_2^y_1 iff
    x_1^y_1-x_1^y_2 >= x_2^y_1-x_2^y^2 iff
    e^(y_1ln(x_1))-e^(y_2ln(x_1)) >= e^(y_1ln(x_2))-e^(y_2ln(x_2)) iff
    (e^((y_1-y_2)ln(x_1))-1)e^(y_2ln(x_1)) >= (e^((y_1-y_2)ln(x_2))-1)e^(y_2ln(x_2)).
    As the logarithm and exponential are increasing and since y_2 is positive:
    x_1 >= x_2 implies
    ln(x_1) >= ln(x_2) implies
    y_2ln(x_1) >= y_2ln(x_2) implies
    e^(y_2ln(x_1)) >= e^(y_2ln(x_2)).
    Then for the other terms, since y_1 >= y_2, we have y_1-y_2 >= 0, so:
    ln(x_1) >= ln(x_2) implies
    (y_1-y_2)ln(x_1) >= (y_1-y_2)ln(x_2) implies
    e^((y_1-y_2)ln(x_1))-1 >= e^((y_1-y_2)ln(x_2))-1
    Finally, ince e^(y_2ln(x_1)) >= e^(y_2ln(x_2)) > 0, we may multiply the two inequalities to obtain the result.

    • @Notthatkindofdr
      @Notthatkindofdr 3 года назад

      Unfortunately this proof has the same flaw as others in the comments. The two sides of your inequality e^((y_1-y_2)ln(x_1))-1 >= e^((y_1-y_2)ln(x_2))-1 (this would be easier to read as (x_1)^(y_1-y_2)-1 >= (x_2)^(y_1-y_2)-1, by the way) could both be negative, and then you cannot multiply it by the other inequality. (To see what I mean, note that -2 > -3 and 5 > 3, but -10 < -9.) This case corresponds to the second "branch" that Michael does in the video.

    • @Notthatkindofdr
      @Notthatkindofdr 3 года назад

      In fact, @Vincent Lefèvre has a counterexample to your generalization: (1/3)^2 + (1/2)^1000 < (1/3)^1000 + (1/2)^2 in the comments below.

    • @charlottedarroch
      @charlottedarroch 3 года назад

      @@Notthatkindofdr Oh yes, you're right. If we change the constraint to x_1 >= x_2 >= ... >= x_n >= 1, then the result holds. But one can certainly do better. Though the details will be more complicated than the rearrangement inequality.

  • @romajimamulo
    @romajimamulo 3 года назад +5

    Homework: cross multiply (which doesn't flip the inequality because the denominators are positive), then algebraically rearrange to (ln(a))/(1-a) ≤ (ln(b))/(1-b)
    Then, notice how this is like "prove that f(x)=(ln(x))/(1-x) is increasing on the range (0,1)"
    Which can then be done by taking the derivative and checking for critical points

    • @fix5072
      @fix5072 3 года назад +2

      Nicely done!
      trying to check for the critical point is not easy with trying to apply algebra, since you'd have to use Lambert W function (x=-W(-1/e)=1), but trying nice values will eventually give you the answer that x>0 except for x=1 which is however not defined for the function ln(x)/(1-x).

    • @oscarengland6122
      @oscarengland6122 3 года назад +1

      To prove that f(x)=(ln(x))/(1-x) is increasing would it be enough to observe that the numerator is increasing and the denominator is decreasing therefore f(x) must be increasing?

    • @romajimamulo
      @romajimamulo 3 года назад +1

      @@oscarengland6122 Yes, on the range (0,1) that does work

  • @tanimahmed8095
    @tanimahmed8095 3 года назад +4

    @Michael Penn sir, your videos are awesome. Can you please make a video on the proof of L' Hospital's Rule?

    • @craftexx15
      @craftexx15 3 года назад +1

      He already did. Look up in his real analysis course playlist.

  • @karangupta1825
    @karangupta1825 3 года назад +4

    Sir, please make some videos on continued fractions, infinite series and telescopic series.

    • @karangupta1825
      @karangupta1825 3 года назад +1

      @@guydror7297 you are saying that to whom
      To me
      Or
      To the YouThoober

  • @nekrozvalkyrus1754
    @nekrozvalkyrus1754 3 года назад

    Does this not work?
    For a=b they are equal
    Suppose b>a
    (b/a)^b *a^b +b^a=b^b +b^a
    (b/a)^a *a^a +b^b=b^b+b^a
    ((b/a)^b -1)a^b +a^b +b^a=b^a +b^b=((b/a)^a -1)a^a +a^a +b^b
    ((b/a)^b -1)a^b>((b/a)^a -1)a^a
    Therefore a^a+b^b>=a^b +b^a
    Edit: I didn't consider the case of a

  • @jakobhablitz1429
    @jakobhablitz1429 3 года назад +13

    I burnt my poptart after getting distracted watching this

    • @prithujsarkar2010
      @prithujsarkar2010 3 года назад +2

      3 weeks ago commented ? huh ?

    • @Tiqerboy
      @Tiqerboy 3 года назад

      If you switched to Kellogg's Frosted Flakes they would have gone soggy in milk. Can't win, even with a tiger on your side.

    • @jakobhablitz1429
      @jakobhablitz1429 3 года назад

      @@prithujsarkar2010 I'm simply built different

    • @prithujsarkar2010
      @prithujsarkar2010 3 года назад

      @@jakobhablitz1429 lmao

    • @FisicoNuclearCuantico
      @FisicoNuclearCuantico 3 года назад

      @Jakob Hablitz 🙁

  • @lawlietl1370
    @lawlietl1370 3 года назад +2

    Is possible obtain a factorization for (a^b) +/- (b^a) ?

  • @dpeastman
    @dpeastman 3 года назад +1

    Here's my much simpler solution:
    Assume a

  • @dneary
    @dneary 3 года назад +3

    I split out the equality, and followed the LHS method myself with a

    • @replicaacliper
      @replicaacliper 3 года назад

      Why does that only work if b-a>=1? x^k is strictly increasing on x>=0 whenever k>0, then a^(b-a)-1=1, then both of these quantities are positive, and since 0

    • @dneary
      @dneary 3 года назад

      @@replicaacliper I need to look back, but... a^b+b^a < a^a+b^b \implies a^b-a^a < b^b-b^a \implies a^a(a^(b-a)-1) < b^a(b^(b-a)-1) and since we assumed 0

  • @Yash-ql7vn
    @Yash-ql7vn 3 года назад +3

    Hey, just discovered your channel through recommendations, thanks for making a great video. Just subscribed! Hoping for more quality content 💛

  • @andimandi8491
    @andimandi8491 3 года назад +1

    I had the conclusion that a^a+b^b=>a^b+b^a because there are always 3 cases: case 1. a=b; case 2. a>b; case 3. ab^a+a^b.

  • @nopenoper1980
    @nopenoper1980 3 года назад

    Given b>a>0
    a^b-a^a

  • @tomatrix7525
    @tomatrix7525 3 года назад +1

    The proof of the final one is quite simple. Here is a rough outline. Given 0

  • @ollllj
    @ollllj 2 года назад

    complex numbers feel left out of all the fun inequality.
    maybe inequality is just incomplete (and not all that useful)

  • @HideyukiWatanabe
    @HideyukiWatanabe 3 года назад

    It holds that a^a + b^b >= 2((a+b)/2)^((a+b)/2) >= a^b + b^a. Use downward convexity of f(x)=x^x and Jensen's inequality implies the former. The latter needs some messy calculus.

  • @purity_one
    @purity_one 3 года назад +3

    Wait, we have proved it only for b≥1 and b∈(0,1) (considering "homework").
    But what about negative values of b? (cuz in the task we suppose b is Rational)
    Or is there something I didn't understand?

    • @davode76166
      @davode76166 3 года назад +4

      Consider R+ in the first line of the problem

    • @Paulo-jh8vz
      @Paulo-jh8vz 3 года назад +2

      b it's a positive real number

    • @purity_one
      @purity_one 3 года назад +4

      Oh, R+ means real positive numbers. Got it, thx

  • @nightmare9566
    @nightmare9566 3 года назад

    a^a + b^b because it ensures that the largest number is raised to the largest exponent. either a is larger and then you have a^a, or b is larger and you get b^b. Not exactly rigorous, but do we really need more than that immediate intuition? 3^4 + 4^3 < 3^3 + 4^4, 100^2 + 2^100 < 100^100 + 2^2 etc.

  • @williammauriciogiraldomuri9855
    @williammauriciogiraldomuri9855 3 года назад

    I think proof for the homework might go like this:
    - lnx

  • @AAA-mv7dv
    @AAA-mv7dv 3 года назад

    Given that a,b are positive real numbers, further suppose that a>b and try to find such values for the inequality a^a + b^b < a^b + b^a and you'll arrive at the conclusion that it's never true for such values.

  • @dacianbonta2840
    @dacianbonta2840 3 года назад +1

    Chapter 14 of Barnard and Child "Higher Algebra" for a calculus-free approach to this topic. Also, the post of kajamix

  • @josephcheng5949
    @josephcheng5949 3 года назад +9

    3:33 that escalated quickly from algebra to calculus! 😱 😂

    • @mtaur4113
      @mtaur4113 3 года назад +1

      If we only look at whole numbers, some other argument might be possible, but it might not be easy without calculus... If we allow non-whole numbers, it was always calculus all along... :)

  • @prathmeshraut1616
    @prathmeshraut1616 3 года назад +2

    Need a like from Micheal penn

  • @rayansoufargi1439
    @rayansoufargi1439 2 года назад

    Ln(x)-ln(a)/x-a is obviously decreasing and
    b

  • @knvcsg1839
    @knvcsg1839 3 года назад +1

    I love the way you deduce the approach from fine details of the problem. Loved it.

  • @mattcassar4749
    @mattcassar4749 3 года назад +4

    Edit: there is a mistake, so this proof only works for b >= 1. I'm not going to extend it to handle the a, b in (0, 1) case, but it shouldn't be too hard to fix it :)
    I think there's a simpler proof, but I might have made a mistake.
    WLOG assume a

    • @Notthatkindofdr
      @Notthatkindofdr 3 года назад

      I had thought of an algebraic solution like this too, but as you say there is flaw when b= a^a(1 - a^(b - a)) + a^a(b^(b - a) - 1) does not follow if b^(b-a)-1

  • @einsteingonzalez4336
    @einsteingonzalez4336 3 года назад +1

    I thought that the problem is similar to a^b > b^a, but this isn't.
    Whoops!

  • @toddbiesel4288
    @toddbiesel4288 3 года назад +1

    So a^b + b^a is never larger?
    Mamma mia!

  • @junkdubious
    @junkdubious Год назад

    Wonder if this easier if you evaluate these as limits.

  • @ayoubshams261
    @ayoubshams261 2 года назад

    thanks for btfl problems and sltns

  • @inquirusinq6060
    @inquirusinq6060 3 года назад

    At 2:42 you write an unequality a^b-a^a = a^b-a^a if b>=a.
    Sry for eng. Not my native ;3

  • @andrewmcnamara3959
    @andrewmcnamara3959 3 года назад +1

    Got lost fairly quickly on this one

  • @alimoharam4362
    @alimoharam4362 2 года назад

    this problem reminds me of a song

  • @mtaur4113
    @mtaur4113 3 года назад

    Let a belong to the interval (0,1).
    Inequality at 10:25 is true for all x in (a,b) if and only if it is true for x = a, so let x = a.
    Let g(t) = [ln(t) - ln(a)]/[t - a]
    We want to prove that -ln(a) < g(t) for all t in (a, 1). t=1 is shown at 12:08.
    If we can show that g'(t) g(1) > -ln(a) for all such t.
    g'(t) can be done by the quotient rule. To solve the inequality g'(t)>0, compute g' and multiply by the denominator, (x-a)^2. Simplify to get:
    1/t < [ln(t) - ln(a)]/(t - a)
    The right-hand side is the average rate of change of ln on the interval (a,t). By the mean value theorem, that is equal to the derivative of ln at some intermediate point, that is, "ln-prime of c" or 1/c, where c is in (a,t)
    So we want to solve for all t such that 1/t < 1/c. This is in fact all t in (a,1), because c < t and both numbers are positive.

  • @donl.3731
    @donl.3731 3 года назад +1

    all im hearing is "m to the b, m to the b, m m m m m to the b"

  • @zhuolovesmath7483
    @zhuolovesmath7483 3 года назад

    let f(x)=(lnx-lna)/(x-a)
    All we need to prove is that f'(x)1+lnx
    then we'll get the answer

  • @piojo1987
    @piojo1987 3 года назад

    The last step on the board can be proved by seeing that the ln function has negative second derivative. The expression (ln(x)-ln(a)/(x-a)) is the avarage of the slope of the ln function between a and x, and because ln has negative second derivative, increasing x with fixed a decreases the result of that average. As x=b is lower than x=1, the average slope from a to b is greater than the average slope of ln from a to 1, the HW inequality is proved. And that’s a good place to stop!

  • @wepped482
    @wepped482 3 года назад

    Yeah, only if you are talking about positive numbers.. which is your suppose statement. Okays.

  • @no_one6749
    @no_one6749 3 года назад

    explaining >= doing

  • @sirlight-ljij
    @sirlight-ljij 3 года назад +1

    This reminded me of a rearrangement inequality, which stated that for all x1,y1,x2,y2>0 x1y1+x2y2>x1y2+x2y1. This generalises to arbitrary number of summands. And what is surprising is that this inequality generalises even to arbitrary functions. If we have x1

    • @Notthatkindofdr
      @Notthatkindofdr 3 года назад +1

      Unfortunately the inequality f1'(x)f2'(1/2).

    • @sirlight-ljij
      @sirlight-ljij 3 года назад

      @@Notthatkindofdr Ah, yes, sorry, we need a^x, not x^a as a function. Then the derivative will be ln a * a^x which is always bigger than ln b * b^x if a>b

    • @Notthatkindofdr
      @Notthatkindofdr 3 года назад

      @@sirlight-ljij Sorry, I don't think that is quite true either. Try a=e^(-.95), b=e^(-6), x=e^(-1), for example. Then a>x>b, but ln(a)a^x=-0.67

    • @sirlight-ljij
      @sirlight-ljij 3 года назад

      @@Notthatkindofdr yeah, sadly it looks like this approach only works for a,b>1

  • @TheUltiCAKE
    @TheUltiCAKE 3 года назад

    Put any number into a and b (as long as they are not the same and not 0) and find out...ez pz

  • @A_Helder16
    @A_Helder16 3 года назад

    Love Your Algebra Series Videos... Can you make a video review of the three parts of Sylow's Theorem

  • @3snoW_
    @3snoW_ 3 года назад

    5:06 How is this not always true? if 0 < a < x < b then both ln(a) < ln(b) and a^x < b^x are true, hence their product must also share the same relationship: ln(a) * a^x < ln(b) * b^x regardless of whether b is greater than 1 or not

    • @subinmdr
      @subinmdr 3 года назад

      I think its because ln a and ln b are negative when b

  • @Giyga
    @Giyga 3 года назад

    My first intuition was 1^1+2^2 or 1^2+2^1

  • @annanemustapha7279
    @annanemustapha7279 3 года назад

    Witch is larger ? ( aⁿ + b) or (a + bⁿ) if a

  • @yeahyeah54
    @yeahyeah54 3 года назад

    obviously a^a +b^b is bigger.
    this is my thought: i took a=1,b=2 and that's all folks

    • @yeahyeah54
      @yeahyeah54 3 года назад

      lol Michael did the same thing, i didn't see the video when i was writing

  • @MrRyanroberson1
    @MrRyanroberson1 3 года назад

    What if... they're negative numners from the set x/(2y+1) for x,y integers?

  • @hq4889
    @hq4889 3 года назад

    Hii Michael please solve this problem 👇
    Find +ve integers a and b such that
    (a^1/3 +b^1/3 -1) ^2 = 49 +20 (6) ^1/3

  • @fatalaf
    @fatalaf 3 года назад

    Those are some long hoodie strings my guy

  • @josephcheng5949
    @josephcheng5949 3 года назад

    OMG @Michael Penn you give homework now?! 😆

  • @daniello4038
    @daniello4038 3 года назад

    The inequality can also be proved by letting b=ax, x>=1.

  • @Nyarmith
    @Nyarmith 3 года назад

    wow, these videos are actually fun

  • @mikegallegos7
    @mikegallegos7 3 года назад

    HOMEWORK !!!
    YAAAAaaaaahhggggghh !!
    LOL

  • @jesusalej1
    @jesusalej1 3 года назад

    Who are those kids? 5:50

  • @שוןאלמליח
    @שוןאלמליח 3 года назад

    Brilliant video

  • @FisicoNuclearCuantico
    @FisicoNuclearCuantico 3 года назад

    Best regards Mike.

  • @gourabjitbiswas
    @gourabjitbiswas 3 года назад

    🔥🔥🔥

  • @TimesOfSilence
    @TimesOfSilence 3 года назад +1

    Let's make a game and count how many times Michael Penn uses for the word "but" in the beginning of a sentence that doesn''t go in a different direction than the sentence before.
    Example: "1+2+3 is equal to 3+3. But that is equal to 6."
    My guess without having seen the whole video yet is 40 times.

  • @random-td7tf
    @random-td7tf 3 года назад +3

    Nice, I see time travellers here.

  • @AcaciaAvenue
    @AcaciaAvenue 3 года назад +1

    I solved it in a different way, please tell me if I did any mistake:
    I need to prove that a^a + b^b >= a^b + b^a.
    Suppose than a>b. I can then write a = b+c, with c>0. I replace this in the above inequality:
    (b+c)^(b+c) + b^b >= (b+c)^b + b^(b+c)
    I bring the (b+c) terms on the left and the rest on the right:
    (b+c)^(b+c) - (b+c)^b >= b^(b+c) - b^b
    I then split the terms with (b+c) on the exponent:
    [(b+c)^b]*[(b+c)^c] - (b+c)^b >= (b^b)*(b^c) - b^b
    I factor (b+c)^b on the left side and b^b on the right side
    [(b+c)^b] * [(b+c)^c - 1] >= b^b * (b^c - 1)
    I divide bot terms for b^b and "merge" the first term
    [(1+c/b)^b] * [(b+c)^c - 1] >= b^c - 1
    Since c>0 and b>0, c/b is >0 as well, so 1+c/b is greater than 1. Any number greater than 1 powered by a number greater than 0 gives a number greater than 1, so (1+c/b)^b is greater than 1.
    Also, since c>0, (b+c)^c is surely greater than b^c, and that stays true if I subtract 1 from both.
    So on the left side I have something bigger than the right side multiplied by something bigger than 1, so the inequality is surely true.

    • @keinKlarname
      @keinKlarname 3 года назад

      @Hekatos: Great solution!

    • @Notthatkindofdr
      @Notthatkindofdr 3 года назад

      Your last step has a flaw, unfortunately. If a(b+c)^c-1>b^c-1, and if both sides of the inequality are negative then you cannot just multiply it by the inequality (1+c/b)^b>1. (For example, -2>-3 and 2>1, but -4

  • @leefisher6366
    @leefisher6366 3 года назад +1

    Well, two squared is four, three cubed is twenty-seven, and that makes thirty one.
    Alternatively, two cubed is eight, and three squared is nine, only giving seventeen.
    In general, if a

  • @sabitasahoo5388
    @sabitasahoo5388 3 года назад +1

    One of the most underrated channel in RUclips.

    • @speedsterh
      @speedsterh 3 года назад

      I love it too, but the entry level math-wise is pretty steep for someone who hasn't studied maths at college

  • @tomdebom1346
    @tomdebom1346 3 года назад

    I understood about 0% of this but thanks

  • @jeetenderkakkar7570
    @jeetenderkakkar7570 3 года назад

    Pakistan

  • @hsjkdsgd
    @hsjkdsgd 3 года назад

    Definitely a hard problem to figure out

  • @donhavlicek
    @donhavlicek 3 года назад

    Stop using "larger"... it's properly "GREATER THAN".